FTC in Integral Equation












0












$begingroup$


I am working on the following exercise (Source is unknown, unfortunately):




Find all continuous, non-negative functions $f:Bbb{R}toBbb{R}$ satisfying
$$int_0^xf(t)dt=(f(x))^alpha +C$$
for some constant $Cneq 0$ and $alpha >1$.




Here is my attempt:
Because of FTC, we know that $(f(x))^alpha$ is a differentiable function (thus continuous), and because the composition of two continuous is also continuous, $f$ is continuous. Applying FTC we have that
begin{align*}
frac{d}{dx}int_0^xf(t)dt & = frac{d}{dx}Big[(f(x))^alpha+CBig]
\ f(x) & =alpha f(x)^{alpha-1}f'(x)
\ f(x)(alpha f(x)^{alpha-2}f'(x)-1) & = 0
end{align*}

so $f(x)=0$ or $f'(x)=alpha f(x)^{2-alpha}$.
Now, because this equation is in the form $y'+p(t)y=0$, we can probably do something with integrating factors and solve this D.E. However, I don't want to use any differential equation knowledge to solve this. How can I proceed?










share|cite|improve this question











$endgroup$












  • $begingroup$
    I don't understand the grammar in your penultimate sentence. Are you saying you don't want to use any differential equation knowledge to solve this? Your stated form for the equation is wrong anyway; I recommend the Ansatz $g=f^beta$, choosing $beta$ so $g^prime$ is $g$-independent.
    $endgroup$
    – J.G.
    Jan 24 at 19:18










  • $begingroup$
    But you already used some differential equation to solve: the very first step !
    $endgroup$
    – DonAntonio
    Jan 24 at 19:18


















0












$begingroup$


I am working on the following exercise (Source is unknown, unfortunately):




Find all continuous, non-negative functions $f:Bbb{R}toBbb{R}$ satisfying
$$int_0^xf(t)dt=(f(x))^alpha +C$$
for some constant $Cneq 0$ and $alpha >1$.




Here is my attempt:
Because of FTC, we know that $(f(x))^alpha$ is a differentiable function (thus continuous), and because the composition of two continuous is also continuous, $f$ is continuous. Applying FTC we have that
begin{align*}
frac{d}{dx}int_0^xf(t)dt & = frac{d}{dx}Big[(f(x))^alpha+CBig]
\ f(x) & =alpha f(x)^{alpha-1}f'(x)
\ f(x)(alpha f(x)^{alpha-2}f'(x)-1) & = 0
end{align*}

so $f(x)=0$ or $f'(x)=alpha f(x)^{2-alpha}$.
Now, because this equation is in the form $y'+p(t)y=0$, we can probably do something with integrating factors and solve this D.E. However, I don't want to use any differential equation knowledge to solve this. How can I proceed?










share|cite|improve this question











$endgroup$












  • $begingroup$
    I don't understand the grammar in your penultimate sentence. Are you saying you don't want to use any differential equation knowledge to solve this? Your stated form for the equation is wrong anyway; I recommend the Ansatz $g=f^beta$, choosing $beta$ so $g^prime$ is $g$-independent.
    $endgroup$
    – J.G.
    Jan 24 at 19:18










  • $begingroup$
    But you already used some differential equation to solve: the very first step !
    $endgroup$
    – DonAntonio
    Jan 24 at 19:18
















0












0








0





$begingroup$


I am working on the following exercise (Source is unknown, unfortunately):




Find all continuous, non-negative functions $f:Bbb{R}toBbb{R}$ satisfying
$$int_0^xf(t)dt=(f(x))^alpha +C$$
for some constant $Cneq 0$ and $alpha >1$.




Here is my attempt:
Because of FTC, we know that $(f(x))^alpha$ is a differentiable function (thus continuous), and because the composition of two continuous is also continuous, $f$ is continuous. Applying FTC we have that
begin{align*}
frac{d}{dx}int_0^xf(t)dt & = frac{d}{dx}Big[(f(x))^alpha+CBig]
\ f(x) & =alpha f(x)^{alpha-1}f'(x)
\ f(x)(alpha f(x)^{alpha-2}f'(x)-1) & = 0
end{align*}

so $f(x)=0$ or $f'(x)=alpha f(x)^{2-alpha}$.
Now, because this equation is in the form $y'+p(t)y=0$, we can probably do something with integrating factors and solve this D.E. However, I don't want to use any differential equation knowledge to solve this. How can I proceed?










share|cite|improve this question











$endgroup$




I am working on the following exercise (Source is unknown, unfortunately):




Find all continuous, non-negative functions $f:Bbb{R}toBbb{R}$ satisfying
$$int_0^xf(t)dt=(f(x))^alpha +C$$
for some constant $Cneq 0$ and $alpha >1$.




Here is my attempt:
Because of FTC, we know that $(f(x))^alpha$ is a differentiable function (thus continuous), and because the composition of two continuous is also continuous, $f$ is continuous. Applying FTC we have that
begin{align*}
frac{d}{dx}int_0^xf(t)dt & = frac{d}{dx}Big[(f(x))^alpha+CBig]
\ f(x) & =alpha f(x)^{alpha-1}f'(x)
\ f(x)(alpha f(x)^{alpha-2}f'(x)-1) & = 0
end{align*}

so $f(x)=0$ or $f'(x)=alpha f(x)^{2-alpha}$.
Now, because this equation is in the form $y'+p(t)y=0$, we can probably do something with integrating factors and solve this D.E. However, I don't want to use any differential equation knowledge to solve this. How can I proceed?







calculus proof-writing






share|cite|improve this question















share|cite|improve this question













share|cite|improve this question




share|cite|improve this question








edited Jan 24 at 19:20







高田航

















asked Jan 24 at 19:13









高田航高田航

1,360418




1,360418












  • $begingroup$
    I don't understand the grammar in your penultimate sentence. Are you saying you don't want to use any differential equation knowledge to solve this? Your stated form for the equation is wrong anyway; I recommend the Ansatz $g=f^beta$, choosing $beta$ so $g^prime$ is $g$-independent.
    $endgroup$
    – J.G.
    Jan 24 at 19:18










  • $begingroup$
    But you already used some differential equation to solve: the very first step !
    $endgroup$
    – DonAntonio
    Jan 24 at 19:18




















  • $begingroup$
    I don't understand the grammar in your penultimate sentence. Are you saying you don't want to use any differential equation knowledge to solve this? Your stated form for the equation is wrong anyway; I recommend the Ansatz $g=f^beta$, choosing $beta$ so $g^prime$ is $g$-independent.
    $endgroup$
    – J.G.
    Jan 24 at 19:18










  • $begingroup$
    But you already used some differential equation to solve: the very first step !
    $endgroup$
    – DonAntonio
    Jan 24 at 19:18


















$begingroup$
I don't understand the grammar in your penultimate sentence. Are you saying you don't want to use any differential equation knowledge to solve this? Your stated form for the equation is wrong anyway; I recommend the Ansatz $g=f^beta$, choosing $beta$ so $g^prime$ is $g$-independent.
$endgroup$
– J.G.
Jan 24 at 19:18




$begingroup$
I don't understand the grammar in your penultimate sentence. Are you saying you don't want to use any differential equation knowledge to solve this? Your stated form for the equation is wrong anyway; I recommend the Ansatz $g=f^beta$, choosing $beta$ so $g^prime$ is $g$-independent.
$endgroup$
– J.G.
Jan 24 at 19:18












$begingroup$
But you already used some differential equation to solve: the very first step !
$endgroup$
– DonAntonio
Jan 24 at 19:18






$begingroup$
But you already used some differential equation to solve: the very first step !
$endgroup$
– DonAntonio
Jan 24 at 19:18












1 Answer
1






active

oldest

votes


















0












$begingroup$

$frac{df}{dx}=alpha f^{2-alpha}$. By separating variables, we get $f^{alpha-2}df=alpha dx$. By integrating both sides, we have $frac{1}{alpha-1}f^{alpha-1}=alpha x+C$. Therefore, $f(x)=(alpha(alpha-1)x+C)^{frac{1}{alpha-1}}$ if $alphaneq1$. When $alpha=1$ then $frac{df}{f}=dx$. Therefore we have $ln f=x+C$. Hence $f(x)=Ce^{x}$.






share|cite|improve this answer









$endgroup$













  • $begingroup$
    If you evaluate the given relation at $x=0$, you can see that the given answer is probably not correct.
    $endgroup$
    – Rigel
    Jan 24 at 19:53











Your Answer





StackExchange.ifUsing("editor", function () {
return StackExchange.using("mathjaxEditing", function () {
StackExchange.MarkdownEditor.creationCallbacks.add(function (editor, postfix) {
StackExchange.mathjaxEditing.prepareWmdForMathJax(editor, postfix, [["$", "$"], ["\\(","\\)"]]);
});
});
}, "mathjax-editing");

StackExchange.ready(function() {
var channelOptions = {
tags: "".split(" "),
id: "69"
};
initTagRenderer("".split(" "), "".split(" "), channelOptions);

StackExchange.using("externalEditor", function() {
// Have to fire editor after snippets, if snippets enabled
if (StackExchange.settings.snippets.snippetsEnabled) {
StackExchange.using("snippets", function() {
createEditor();
});
}
else {
createEditor();
}
});

function createEditor() {
StackExchange.prepareEditor({
heartbeatType: 'answer',
autoActivateHeartbeat: false,
convertImagesToLinks: true,
noModals: true,
showLowRepImageUploadWarning: true,
reputationToPostImages: 10,
bindNavPrevention: true,
postfix: "",
imageUploader: {
brandingHtml: "Powered by u003ca class="icon-imgur-white" href="https://imgur.com/"u003eu003c/au003e",
contentPolicyHtml: "User contributions licensed under u003ca href="https://creativecommons.org/licenses/by-sa/3.0/"u003ecc by-sa 3.0 with attribution requiredu003c/au003e u003ca href="https://stackoverflow.com/legal/content-policy"u003e(content policy)u003c/au003e",
allowUrls: true
},
noCode: true, onDemand: true,
discardSelector: ".discard-answer"
,immediatelyShowMarkdownHelp:true
});


}
});














draft saved

draft discarded


















StackExchange.ready(
function () {
StackExchange.openid.initPostLogin('.new-post-login', 'https%3a%2f%2fmath.stackexchange.com%2fquestions%2f3086253%2fftc-in-integral-equation%23new-answer', 'question_page');
}
);

Post as a guest















Required, but never shown

























1 Answer
1






active

oldest

votes








1 Answer
1






active

oldest

votes









active

oldest

votes






active

oldest

votes









0












$begingroup$

$frac{df}{dx}=alpha f^{2-alpha}$. By separating variables, we get $f^{alpha-2}df=alpha dx$. By integrating both sides, we have $frac{1}{alpha-1}f^{alpha-1}=alpha x+C$. Therefore, $f(x)=(alpha(alpha-1)x+C)^{frac{1}{alpha-1}}$ if $alphaneq1$. When $alpha=1$ then $frac{df}{f}=dx$. Therefore we have $ln f=x+C$. Hence $f(x)=Ce^{x}$.






share|cite|improve this answer









$endgroup$













  • $begingroup$
    If you evaluate the given relation at $x=0$, you can see that the given answer is probably not correct.
    $endgroup$
    – Rigel
    Jan 24 at 19:53
















0












$begingroup$

$frac{df}{dx}=alpha f^{2-alpha}$. By separating variables, we get $f^{alpha-2}df=alpha dx$. By integrating both sides, we have $frac{1}{alpha-1}f^{alpha-1}=alpha x+C$. Therefore, $f(x)=(alpha(alpha-1)x+C)^{frac{1}{alpha-1}}$ if $alphaneq1$. When $alpha=1$ then $frac{df}{f}=dx$. Therefore we have $ln f=x+C$. Hence $f(x)=Ce^{x}$.






share|cite|improve this answer









$endgroup$













  • $begingroup$
    If you evaluate the given relation at $x=0$, you can see that the given answer is probably not correct.
    $endgroup$
    – Rigel
    Jan 24 at 19:53














0












0








0





$begingroup$

$frac{df}{dx}=alpha f^{2-alpha}$. By separating variables, we get $f^{alpha-2}df=alpha dx$. By integrating both sides, we have $frac{1}{alpha-1}f^{alpha-1}=alpha x+C$. Therefore, $f(x)=(alpha(alpha-1)x+C)^{frac{1}{alpha-1}}$ if $alphaneq1$. When $alpha=1$ then $frac{df}{f}=dx$. Therefore we have $ln f=x+C$. Hence $f(x)=Ce^{x}$.






share|cite|improve this answer









$endgroup$



$frac{df}{dx}=alpha f^{2-alpha}$. By separating variables, we get $f^{alpha-2}df=alpha dx$. By integrating both sides, we have $frac{1}{alpha-1}f^{alpha-1}=alpha x+C$. Therefore, $f(x)=(alpha(alpha-1)x+C)^{frac{1}{alpha-1}}$ if $alphaneq1$. When $alpha=1$ then $frac{df}{f}=dx$. Therefore we have $ln f=x+C$. Hence $f(x)=Ce^{x}$.







share|cite|improve this answer












share|cite|improve this answer



share|cite|improve this answer










answered Jan 24 at 19:29









NotoriousJuanGNotoriousJuanG

843




843












  • $begingroup$
    If you evaluate the given relation at $x=0$, you can see that the given answer is probably not correct.
    $endgroup$
    – Rigel
    Jan 24 at 19:53


















  • $begingroup$
    If you evaluate the given relation at $x=0$, you can see that the given answer is probably not correct.
    $endgroup$
    – Rigel
    Jan 24 at 19:53
















$begingroup$
If you evaluate the given relation at $x=0$, you can see that the given answer is probably not correct.
$endgroup$
– Rigel
Jan 24 at 19:53




$begingroup$
If you evaluate the given relation at $x=0$, you can see that the given answer is probably not correct.
$endgroup$
– Rigel
Jan 24 at 19:53


















draft saved

draft discarded




















































Thanks for contributing an answer to Mathematics Stack Exchange!


  • Please be sure to answer the question. Provide details and share your research!

But avoid



  • Asking for help, clarification, or responding to other answers.

  • Making statements based on opinion; back them up with references or personal experience.


Use MathJax to format equations. MathJax reference.


To learn more, see our tips on writing great answers.




draft saved


draft discarded














StackExchange.ready(
function () {
StackExchange.openid.initPostLogin('.new-post-login', 'https%3a%2f%2fmath.stackexchange.com%2fquestions%2f3086253%2fftc-in-integral-equation%23new-answer', 'question_page');
}
);

Post as a guest















Required, but never shown





















































Required, but never shown














Required, but never shown












Required, but never shown







Required, but never shown

































Required, but never shown














Required, but never shown












Required, but never shown







Required, but never shown







Popular posts from this blog

Can a sorcerer learn a 5th-level spell early by creating spell slots using the Font of Magic feature?

Does disintegrating a polymorphed enemy still kill it after the 2018 errata?

A Topological Invariant for $pi_3(U(n))$